Đến nội dung

lahantaithe99 nội dung

Có 878 mục bởi lahantaithe99 (Tìm giới hạn từ 12-05-2020)



Sắp theo                Sắp xếp  

#558781 CMR: $\sum \sqrt{\frac{a^{2}}...

Đã gửi bởi lahantaithe99 on 11-05-2015 - 20:13 trong Bất đẳng thức và cực trị

 

Cho a,b,c > 0 và $a^{2}+b^{2}+c^{2}=3$. CMR:

$\sum \sqrt{\frac{a^{2}}{a^{2}+b+c}} \leqslant \sqrt{3}$

 

 

 

Đặt biểu thức vế trái là $A$ . Áp dụng BĐT Cauchy Schwarz kết hợp với những BĐT sau:

 

1. $a+b+c\leq a^2+b^2+c^2=3$

 

2. $(a^2+b+c)(1+b+c)\geq (a+b+c)^2$ ta có

 

$A^2\leq (a+b+c)\left [ \sum \frac{a}{a^2+b+c} \right ]\leq (a+b+c)\left [ \sum \frac{a(1+b+c)}{(a+b+c)^2} \right ]$

 

$\Leftrightarrow A^2\leq \frac{a+b+c+2(ab+bc+ac)}{a+b+c}\leq \frac{(a+b+c)^2}{a+b+c}=a+b+c\leq 3$

 

$\rightarrow A\leq \sqrt{3}$ (đpcm)




#558615 CMR: $\sum \frac{a^{3}}{2a^{2...

Đã gửi bởi lahantaithe99 on 10-05-2015 - 15:11 trong Bất đẳng thức - Cực trị



Cho a,b,c là các số thực. CMR: $\sum \frac{a^{3}}{2a^{2}-ab+3b^{2}} \geqslant \frac{a+b+c}{4}$

 

BĐT như thế này thì tự hiểu nó là nguyên dương!

 

Đặt biểu thức vế trái là $A$ thì áp dụng Cauchy Schwarz ta có

 

$A=\sum \frac{a^4}{2a^3-a^2b+3ab^2}\geq \frac{(a^2+b^2+c^2)^2}{2(a^3+b^3+c^3)-\sum a^2b+3\sum ab^2}\geq \frac{a+b+c}{4}$

 

$\Leftrightarrow 4(a^2+b^2+c^2)^2\geq (a+b+c)\left [ 2(a^3+b^3+c^3)-\sum a^2b+3\sum ab^2 \right ]$

 

$\Leftrightarrow 2(a^2+b^2+c^2)^2\geq \sum a^3b+5\sum ab^3$ $(*)$

 

Có BĐT phụ quen thuộc sau: $(a^2+b^2+c^2)^2\geq 3(a^3b+b^3c+c^3a)$ hoặc $(a^2+b^2+c^2)^2\geq 3(ab^3+bc^3+ca^3)$

 

Sử dụng BĐT trên ta có ngay $(*)$ nên ta có đpcm.

 

----------------------------------------------------------------------------

Chứng minh BĐT phụ

 

Áp dụng bất đẳng thức $(x+y+z)^2\geq 3(xy+yz+xz)$ thì đặt $(x,y,z)=(a^2+bc-ab,b^2+ac-bc,c^2+ab-ac)$ ta có ngay đpcm




#558599 ĐỀ THI OLYMPIC CHUYÊN KHOA HỌC TỰ NHIÊN NĂM 2015

Đã gửi bởi lahantaithe99 on 10-05-2015 - 11:46 trong Thi HSG cấp Tỉnh, Thành phố. Olympic 30-4. Đề thi và kiểm tra đội tuyển các cấp.

Ngày thi thứ hai

 

Câu IV. Cho $a,b\in\mathbb{Z}$, $n\in\mathbb{Z^+}$. CMR

$A=b^{n-1}a(a+b)(a+2b)...[a+(n-1)b]$ chia hết cho $n!$

 

Câu V. Cho tứ giác $ABCD$ nội tiếp $(O)$. Gọi $I,J$ lần lượt là tâm nội tiếp các tam giác $BAD,CAD$. Gọi $DI,AJ$ lần lượt cắt $(O)$ tại $S,T$. Đường thẳng $IJ$ cắt $AB,CD$ tại $M,N$.

a) Chứng minh rằng $SM,TN$ cắt nhau trên đường tròn $(O)$

b) Gọi đường tròn ngoại tiếp tam giác $ABN$ cắt $CD$ tại $P$ khác $N$. $(CDM)$ cắt $AB$ tại $Q$ khác $M$. Chứng minh rằng $PQ$ đi qua tâm nội tiếp hai tam giác $ABC$ và $DBC$

 

Câu VI. Cho $x,y,z>0$ và $xy+yz+xz=1$. CMR

$\frac{x}{\sqrt{yz}+\sqrt{3}}+\frac{y}{\sqrt{xz}+\sqrt{3}}+\frac{z}{\sqrt{xy}+\sqrt{3}}\leq \frac{1}{4\sqrt{3}xyz}$

 

---------------------------------------------------------------------------

P/s: Rớt rụng răng :))




#558433 ĐỀ THI OLYMPIC CHUYÊN KHOA HỌC TỰ NHIÊN NĂM 2015

Đã gửi bởi lahantaithe99 on 09-05-2015 - 11:07 trong Thi HSG cấp Tỉnh, Thành phố. Olympic 30-4. Đề thi và kiểm tra đội tuyển các cấp.

ĐỀ THI OLYMPIC CHUYÊN KHOA HỌC TỰ NHIÊN NĂM 2015

Môn thi: Toán

 

Ngày thi thứ nhất

 

Câu I: Tìm tất cả các số nguyên tố $p$ sao cho $3^p+4^p$ là số chính phương

 

Câu II. Cho tam giác $ABC$ tâm nội tiếp $(I)$ và $AI$ cắt $BC$ tại $D$. Một đường thẳng đi qua $A$ cắt đường tròn ngoại tiếp tam giác $IBC$ tại $P,Q$ sao cho $P$ nằm giữa $A,Q$.

a) CMR tích $DP.DQ$ không đổi khi $P,Q$ thay đổi

b) Giả sử đoạn thẳng $PQ$ cắt đoạn thẳng $BD$. Trên đoạn $DB$ lấy điểm $M$ sao cho $DM=DP$. Lấy $R$ đối xứng $M$ qua trung điểm $BC$. $(ADR)$ cắt $(IBC)$ tại $S,T$ . $ST$ cắt $BC$ tại $N$. CMR tam giác $DNQ$ cân.

 

Câu III. Hai bạn An và Bình chơi một trò chơi trên bảng vuông kích thước $3\times 2015$ ( $3$ hàng và $2015$ cột) . Hai người chơi lần lượt, An đi trước. Mỗi lần chơi, An đặt vào bảng một hình chữ nhật ngang $1\times 3$ và Bình đặt vào bảng một hình chữ nhật dọc $3\times 1$. Các hình chữ nhật được đặt vào không được chồng lên nhau. Ai đến lượt mình mà không đặt được hình chữ nhật là thua. Giả sử rằng cả hai bạn đều chơi rất giỏi. Hỏi ai có chiến thuật để chắc chắn dành được chiến thắng? 




#556167 $a^{2}+b^{2}+c^{2}+\frac{a+b+c...

Đã gửi bởi lahantaithe99 on 25-04-2015 - 05:08 trong Bất đẳng thức và cực trị

Cho các số dương a, b, c nhỏ hơn 1 thỏa mãn $(1-a)(1-b)(1-c)=abc$

Chứng minh: 

$a^{2}+b^{2}+c^{2}+\frac{a+b+c}{2}\geq \frac{3}{2}$

 

$(\frac{1-a}{a},\frac{1-b}{b},\frac{1-c}{c})=(x,y,z)\Rightarrow (a,b,c)=(\frac{1}{x+1},\frac{1}{y+1},\frac{1}{z+1})$ với $xyz=1$

 

Ta cần chứng minh $\sum (\frac{1}{x+1})^2+\frac{1}{2}\sum \frac{1}{x+1}\geq \frac{3}{2}$

 

Có 1 BĐT quen thuộc là $\frac{1}{(x+1)^2}+\frac{1}{(y+1)^2}\geq \frac{1}{xy+1}=\frac{z}{z+1}$

 

Theo nguyên lí Dirichle giả sử $(x-1)(y-1)\geq 0$ thì $x+y\leq xy+1$ nên

 

$\frac{1}{x+1}+\frac{1}{y+1}\geq\frac{4}{x+y+2}\geq\frac{4}{xy+3}=\frac{4z}{1+3z}$

 

Do đó cần chứng minh $\frac{1}{(z+1)^2}+\frac{z}{2(z+1)}+\frac{2z}{1+3z}\geq 1\Leftrightarrow \frac{z(z-1)^2}{(z+1)^2(1+3z)}\geq 0$ ( đúng)

 

Nên ta có đpcm

 




#550656 tìm nghiệm nguyên $2^a.3^b-5^x.7^y=1$

Đã gửi bởi lahantaithe99 on 01-04-2015 - 02:20 trong Số học

tìm các bộ $(a,b,x,y)\in \mathbb{N}^*$ thỏa

$2^a.3^b-5^x.7^y=1$

 

Ta có $2^a3^b-1=5^x7^y$ nên $5|2^a3^b-1$ $(1)$ và $7|2^a3^b-1$ $(2)$

 

  Xét modun $4$ cho $a,b$ ta suy ra được rằng $a,b$ phải cùng tính chẵn lẻ mới thỏa mãn $(1)$

 

Xét modun $6$ cho $a,b$ ta suy ra $(a,b)$ có các dạng là

 

$(6k,6m), (6k+1,6m+4),(6k+2,6m+2), (6k+3,6m), (6k+4,6m+4), (6k+5,6m+2)$ 

 

để thỏa mãn $(2)$

 

Như vậy suy ra để thỏa mãn cả $(1)$ và $(2)$ thì $a,b$ phải cùng chẵn. Đặt $a=2c,b=2d$ ( $c,d\in\mathbb{N^*}$)

 

Phương trình trở thành $5^x7^y=(2^c3^d-1)(2^c3^d+1)$

 

Đến đây chắc là xét phương trình tích   :D




#547228 CMR $a^2+2b^2\vdots p\Leftrightarrow$ $a$ và...

Đã gửi bởi lahantaithe99 on 14-03-2015 - 23:31 trong Số học

$1,$ cho $a,b$ nguyên và $p$ là số nguyên tố có dạng $8k+5(k\in \mathbb{N})$

CMR $a^2+2b^2\vdots p\Leftrightarrow \left\{\begin{matrix} a\vdots p\\b\vdots p \end{matrix}\right.$

 

NTP

Bài 1:

 

Giả sử $a,b$ không chia hết cho $p$ tức là $(a,p)=(b,p)=1$

 

Áp dụng định lý Fermat có $\left\{\begin{matrix} a^{p-1}= (a^4)^{2k+1}\equiv 1 (mod p) & \\ (4b^4)^{2k+1}\equiv 4^{2k+1} (mod p) & \end{matrix}\right.\Rightarrow 4^{2k+1}-1\equiv (a^2+2b^2)(...)\equiv 0 (mod p)$

 

Hay $2^{\frac{p-1}{2}}\equiv 1$ (mod $p$ ) $(1)$

 

Theo tiêu chuẩn Euler thì $2^{\frac{p-1}{2}}\equiv \left ( \frac{2}{p} \right )$ (mod $p$ )

 

Mà do $p$ là số nguyên tố có dạng $8k+5$ nên $\left ( \frac{2}{p} \right )=(-1)^{\frac{p^2-1}{8}}\equiv -1$ (mod $p$ ) $(2)$

 

Từ $(1)$ và $(2)$ ta thấy mâu thuẫn nên suy ra đpcm




#546491 $xy+yz+zx\leq 8$

Đã gửi bởi lahantaithe99 on 27-02-2015 - 02:10 trong Bất đẳng thức và cực trị

 

Bài 2: Cho các số thực x,y,z thỏa mãn: $\left\{\begin{matrix} x^{2}+xy+y^{2}=3& & \\ y^{2}+yz+z^{2}=16& & \end{matrix}\right.$ .

Chứng minh rằng: $xy+yz+zx\leq 8$ .

 

Bài 3: Cho x,y,z là các số thực dương thay đổi. Tìm GTNN của biểu thức: 

$P=\frac{x^{2}y}{z^{3}}+\frac{y^{2}z}{x^{3}}+\frac{z^{2}x}{y^{3}}+\frac{4xyz}{xy^{2}+yz^{2}+zx^{2}}$ .

 

Bài 2: Bạn xem tại

 

http://diendantoanho...bccaleqslant-8/

 

Bài 3:

 

Đặt $(\frac{x}{z},\frac{y}{x},\frac{z}{y})=(a,b,c)$ suy ra $abc=1$

 

Khi đó cần tìm min $P=\frac{a^2}{c}+\frac{b^2}{a}+\frac{c^2}{b}+\frac{4}{ab+bc+ac}$

 

Áp dụng Cauchy Schwarz thì

 

$P\geq a+b+c+\frac{4}{ab+bc+ac}\geq\sqrt{3(ab+bc+ac)}+\frac{4}{ab+bc+ac}$

 

Bằng $AM-GM$ kết hợp với $ab+bc+ac\geq 3$ ta có

 

$9.\frac{\sqrt{3(ab+bc+ac)}}{9}+4.\frac{1}{ab+bc+ac}\geq 13\sqrt[13]{\frac{1}{9^9(ab+bc+ac)^4}.\sqrt{3^9(ab+bc+ac)^9}}\geq \frac{13}{3}$

 

Do đó $P_{min}=\frac{13}{3}$




#546461 $\frac{a+b+c}{3}\geq \sqrt[11]{...

Đã gửi bởi lahantaithe99 on 26-02-2015 - 22:51 trong Bất đẳng thức - Cực trị

Cho a,b,c là các số thực dương thỏa mãn $abc=1$. Chứng minh:

     $\frac{a+b+c}{3}\geq \sqrt[11]{\frac{a^3+b^3+c^3}{3}}$

 

Đại khái là ta có một BĐT phụ như sau:

 

Cho $a,b,c>0$ thì $5(a+b+c)^6\geq 729abc(a^3+b^3+c^3+2abc)$

 

Áp dụng BĐT trên vào bài toán và dựa vào đk $abc=1$ ta có $5(a+b+c)^6\geq 729(a^3+b^3+c^3+2)$

 

$\Leftrightarrow\frac{a+b+c}{3}\geq \sqrt[6]{\frac{a^3+b^3+c^3+2}{5}}$

 

Áp dụng bất đẳng thức $AM-GM$ thì 

 

$\frac{a^3+b^3+c^3}{3}+\frac{a^3+b^3+c^3}{3}+\frac{a^3+b^3+c^3}{3}+1+1\geq 5\sqrt[5]{\frac{(a^3+b^3+c^3)^3}{27}}$

 

Do đó mà ta thu được $\frac{a+b+c}{3}\geq \sqrt[10]{\frac{a^3+b^3+c^3}{3}}$

 

Mặt khac với $abc=1$ ta dễ thấy $\sqrt[10]{\frac{a^3+b^3+c^3}{3}}\geq \sqrt[11]{\frac{a^3+b^3+c^3}{3}}$

 

Do đó ta có đpcm 




#546356 Tìm Min $\frac{a^2}{b^2+(a+b)^2}+\frac...

Đã gửi bởi lahantaithe99 on 26-02-2015 - 18:42 trong Bất đẳng thức và cực trị

Tìm GTNN của BT 

P=$\frac{a^2}{b^2+(a+b)^2}+\frac{b^2}{a^2+(a+b)^2)}$

 

Cauchy Shwarz ta có

 

$P\geq \frac{(a^2+b^2)^2}{2a^2b^2+(a+b)^2(a^2+b^2)}$

 

Với mọi số thực $a,b$ thì ta luôn có $2a^2b^2\leq \frac{(a^2+b^2)^2}{2}$ và $(a+b)^2\leq 2(a^2+b^2)$

 

Do đó $P_{min}=\frac{2}{5}$




#545890 Chứng minh rằng: $0\leqslant a\leqslant 1\leqslant b...

Đã gửi bởi lahantaithe99 on 24-02-2015 - 17:05 trong Bất đẳng thức và cực trị

Cho các số thực a, b, c thỏa mãn : $a\leqslant b\leqslant c$, a+b+c=6, ab+bc+ca=9.

Chứng minh rằng: $0\leqslant a\leqslant 1\leqslant b\leqslant 3\leqslant c\leqslant 4$

Với $a$ âm thì bài toán vẫn đúng nhé

 

Giả thiết suy ra $(a-b)(a-c)\geq 0\Leftrightarrow a^2-ac-ab+bc\geq 0\Leftrightarrow a^2+9\geq 2a(b+c)=2a(6-a)$

 

hay $(a-1)(a-3)\geq 0$. Nếu $a\geq 3$ thì $a+b+c\geq 9$ (vô lý) nên $a\leq 1$

 

Bằng một cách tương tự có $(b-a)(b-c)\leq 0$ ta suy ra $(b-1)(b-3)\leq 0$ tương đương $1\leq b\leq c$

 

và $(c-1)(c-3)\geq 0$. Nếu $c\leq 1$ thì vô lý nên $c\geq 3$

 

Lại có $c(6-c)=c(a+b)=9-ab\geq 9-\frac{(a+b)^2}{4}=9-\frac{(6-c)^2}{4}$

 

Giải bất phương trình này thu được $c\leq 4$

 

Từ các điều trên suy ra đpcm




#544938 $$\dfrac{1}{b^2+c^2+2}+\dfrac{1...

Đã gửi bởi lahantaithe99 on 20-02-2015 - 02:07 trong Bất đẳng thức và cực trị

Cho các số thực không âm $a,b,c$ thỏa mãn $a+b+c=3$. Chứng minh rằng:

$$\dfrac{1}{b^2+c^2+2}+\dfrac{1}{c^2+a^2+2}+\dfrac{1}{a^2+b^2+2}\leqslant \dfrac{3}{4}$$

 

Khai bút mùng 2 :))

 

BĐT cần chứng minh $\sum \frac{b^2+c^2}{b^2+c^2+2}\geqslant \frac{3}{2}\Leftrightarrow \sum \frac{(b+c)^2+(b-c)^2}{b^2+c^2+2}\geqslant 3$

 

Áp dụng BĐT Cauchy Schwarz ta có

 

$\sum \frac{(b+c)^2}{b^2+c^2+2}\geqslant \frac{4(a+b+c)^2}{2(a^2+b^2+c^2)+6}$ và $\sum \frac{(b-c)^2}{b^2+c^2+2}\geqslant \frac{[(a-b)+(b-c)+(a-c)]^2}{2(a^2+b^2+c^2)+6}=\frac{4(a-c)^2}{2(a^2+b^2+c^2)+6}$

 

Ta cần chứng minh

 

$\frac{4(a+b+c)^2+4(a-c)^2}{2(a^2+b^2+c^2)+6 }\geqslant 3\Leftrightarrow (a+b+c)^2+2(a-c)^2\geqslant 3(a^2+b^2+c^2)$

 

$\Leftrightarrow ab+bc-ac-b^2\geqslant 0\Leftrightarrow (a-b)(b-c)\geqslant 0$ $(*)$

 

Đến đây ta chỉ cần giả sử $a\geqslant b\geqslant c$ thì BĐT $(*)$ luôn đúng nên ta có đpm

 

Dấu $=$ khi $a=b=c=1$




#544692 Chứng minh $\Sigma$$\sqrt{a}$$...

Đã gửi bởi lahantaithe99 on 17-02-2015 - 19:03 trong Bất đẳng thức và cực trị

Cho các số thực dương a, b, c và a+b+c$\leq$3.

Chứng minh $\Sigma$$\sqrt{a}$$\geq$$\Sigma$ab

Áp dụng BĐT $AM-GM$ ta có

  

$\sqrt{a}+\sqrt{a}+a^2\geqslant 3a$

 

Tương tự với các biến còn lại  và cộng theo vế thu được

 

$2(\sqrt{a}+\sqrt{b}+\sqrt{c})+a^2+b^2+c^2\geqslant 3(a+b+c)\geqslant (a+b+c)^2$

 

Rút gọn ta thu đc đpcm




#542918 Vinh danh các thành viên VMF có giải trong kì thi VMO

Đã gửi bởi lahantaithe99 on 04-02-2015 - 02:33 trong Thi HSG Quốc gia và Quốc tế

Nhì còn anh Nguyễn Đức Minh ( Whatever2507 ) nữa anh ơi  :P

 

Cảm ơ chú, anh đã thêm :D .




#542917 Giải phương trình nghiệm nguyên dương: $2^x=1+3^y.7^z$

Đã gửi bởi lahantaithe99 on 04-02-2015 - 02:06 trong Số học

Giải phương trình nghiệm nguyên dương: $2^x=1+3^y.7^z$

 

Với $y\in\mathbb{N^*}$ thì $3|2^x-1$ nên $\Rightarrow x=2t$ ($t\in \mathbb{N^*}$)

 

Khi đó $(2^t-1)(2^t+1)=3^y7^z$. Đặt $2^t-1=3^a7^b,2^t+1=3^c7^d$ ( $a,b,c,d\in\mathbb{N}$)

 

$2=3^c7^d-3^a7^b$ không chia hết cho $3$ và $7$ nên ta có $2$ trường hợp:

 

+) TH1: $c=0,b=0$ thì $2=7^d-3^a\equiv 1$ (mod 3) ( với $a,d$ nguyên dương) (vô lý)

 

+) TH2: $a=0$ dễ thấy $d=0$. Khi đó $2+7^b=3^c$ $(*)$

 

$(*)$ : Xét $c\geqslant 3$

 

Khi đó $7^b\equiv 25$ (mod 27) nên $b\equiv 4$ (mod 9). Khi đó $3^c=2+7^{9k+4}\equiv 35$ (mod 37)

 

Ta thấy nếu $c$ có dạng $6m+1,6m+3,6m+5$ thì $3^c-2$ không chia hết cho $7$ nên $c=2c_1$

 

Khi đó $9^{c_1}\equiv 35$ (mod 37). Xét modul $9$ cho $c_1$ ta thấy $9^{c_1}\not\equiv 35$ (mod 37) nên loại

 

Do đó $c=2$. Từ đó thu được $(x,y,z)=(6,2,1)$




#542796 $\left\{\begin{matrix} 3xy=2(x+y) &...

Đã gửi bởi lahantaithe99 on 03-02-2015 - 01:59 trong Phương trình, hệ phương trình và bất phương trình

Giải hệ phương trình: 

$\left\{\begin{matrix} 3xy=2(x+y) & \\ 5yz=6(y+z) & \\ 4zx=3(z+x) & \end{matrix}\right.$

 

Hệ phương trình tương đương

 

$\left\{\begin{matrix} \frac{1}{x}+\frac{1}{y}=\frac{3}{2} & & \\ \frac{1}{y}+\frac{1}{z}=\frac{5}{6} & & \\ \frac{1}{z}+\frac{1}{x}=\frac{4}{3} & & \end{matrix}\right.$

 

Đến đây giải như bình thường 




#542442 $\sqrt{a}+\sqrt{b}+\sqrt{c}...

Đã gửi bởi lahantaithe99 on 31-01-2015 - 17:59 trong Bất đẳng thức và cực trị

Cho a,b,c >0, a+b=c=3. Chứng minh:

$\sqrt{a}+\sqrt{b}+\sqrt{c} \ge ab+bc+ca$

 

Áp dụng bất đẳng thức $AM-GM$

 

$\sqrt{a}+\sqrt{a}+a^2\geqslant 3a$

 

Thiết lập tương tự với $b,c$ và cộng các vế BĐT lại thu được

 

$2(\sqrt{a})+\sqrt{b}+\sqrt{c})+a^2+b^2+c^2\geqslant 3(a+b+c)=(a+b+c)^2$

 

Rút gọn ta được đpcm

 

Dấu $=$ khi $a=b=c=1$




#542423 $\sum \frac{a^2(b+c)}{b^2+bc+c^2}\geq...

Đã gửi bởi lahantaithe99 on 31-01-2015 - 15:15 trong Bất đẳng thức - Cực trị

  Cho các số thực dương  $a,b,c> 0$ .CMR:

 

   $\frac{a^2(b+c)}{b^2+bc+c^2}+\frac{b^2(c+a)}{c^2+ac+a^2}+\frac{c^2(a+b)}{a^2+ab+b^2}\geq \frac{2(ab+bc+ac)}{a+b+c}$

 

BĐT cần chứng minh tương đương với

 

$\sum \frac{a^2(b+c)(a+b+c)}{b^2+bc+c^2}\geqslant 2(ab+bc+ac)$

 

$\Leftrightarrow a^2+b^2+c^2+\sum \frac{a^2(ab+bc+ac)}{b^2+bc+c^2}\geqslant 2(ab+bc+ac)$

 

Dễ thấy  $a^2+b^2+c^2\geqslant ab+bc+ac$ nên cần chứng minh

 

$\sum \frac{a^2(ab+bc+ac)}{b^2+bc+c^2}\geqslant ab+bc+ac\Leftrightarrow \sum \frac{a^2}{b^2+bc+c^2}\geqslant 1$ $(*)$

 

BĐT $(*)$ luôn đúng khi áp dụng Cauchy Schwarz

 

$\sum \frac{a^2}{b^2+bc+c^2}=\sum \frac{a^4}{a^2b^2+a^2bc+a^2c^2}\geqslant \frac{(a^2+b^2+c^2)^2}{2(a^2b^2+b^2c^2+c^2a^2)+abc(a+b+c)}\geqslant 1$

 

Vậy ta có đpcm




#542358 $ \frac{{\sum {\sqrt {ab - 1} } }}{{\sqrt[4]{{abc}}}...

Đã gửi bởi lahantaithe99 on 30-01-2015 - 17:46 trong Bất đẳng thức - Cực trị

Cho $a,b,c\geqslant 1$ thoả mãn $a+b+c+2=abc$. CMR

 

$\frac{\sqrt{ab-1}+\sqrt{bc-1}+\sqrt{ca-1}}{\sqrt[4]{abc}}\leqslant \frac{1}{2}(\sqrt{a}+\sqrt{b}+\sqrt{c})\sqrt{\frac{1}{\sqrt{a}}+\frac{1}{\sqrt{b}}+\frac{1}{\sqrt{c}}}$




#542263 $2 + 3^{x} = 5^{y}$

Đã gửi bởi lahantaithe99 on 29-01-2015 - 17:49 trong Số học

Tìm x,y nguyên dương sao cho: $2 + 3^{x} = 5^{y}$

 

+) Nếu $x=1$ thì $y=1$

 

+) Nếu $x\geq 2$ thì $5^y\equiv 2$ (mod 9) nên $y=6k+5$ ( $k\in\mathbb{N}$)

 

Khi đó $3^x=5^{6k+5}-2\equiv 23$ (mod 31) $(1)$

 

Mặt khác $3^x=5^y-2\equiv 3$ (mod 4) nên $x$ lẻ

 

Từ đó kết hợp với $(1)$ thì $x=30t+1,30t+3,.....,30t+29$ ( $t$ là số tự nhiên bất kỳ)

 

Khi đó $3^x\equiv 3,27,26,17,29,13,24,30,22,12,15,11,6\not\equiv 23$ (mod 31) nên trái với $(1)$

 

Vậy  $(x,y)=(1,1)$




#542228 $2^{x}+1=3^{y}$

Đã gửi bởi lahantaithe99 on 29-01-2015 - 02:34 trong Đại số

Bài 1: Tìm nghiệm nguyên dương của pt

 

b, $2^{x}+1=3^{y}$

Bài 2: tìm nghiệm nguyên của pt

a, $x^{2}+y^{2}=1999$

b, $x_1^{3}+x_2^{3}+x_3^{3}=304*1975*195*1995$ với $x_1*x_2*x_3$ không chia hết cho 3

c, $\frac{5}{3}x-y=\sqrt{3x+2}-\sqrt{2y-1}-1$

Bài 1: 

 

b) $2^x+1=3^y\equiv 0$ (mod 3) nên $x$ lẻ

 

+) Nếu $x\geq 4$. Khi đó $3^y\equiv 1$ (mod 16)

 

Xét các TH $y=4k,4k+1,4k+2,4k+3$ ( $k\in\mathbb{N}$) ta thấy $y=4k$ ( $k\neq 0$) là thỏa mãn

 

Khi đó $2^x=3^{4k}-1\equiv 0$ (mod 5) (vô lý)

 

Do đó $x<4$. Mà $x$ lẻ nên $x=1,3$. Thế vào tìm $y$

 

Bài 2:

 

a) Một số chính phương chia $4$ có dư là $0,1$ nên $x^2+y^2\equiv 0,1,2$ (mod 4)

 

Mà $1999\equiv 3$ (mod 4) nên phương trình vô nghiệm

 

b) Đặt $(x_1,x_2,x_3)=(a,b,c)$ ( cho dễ viết ) :D

 

Khi đó từ điều kiện đề bài suy ra $a^3+b^3+c^3-3abc=(a+b+c)^3-3(a+b+c)(ab+bc+ac)$ chia hết cho $3$ nên $a+b+c$ chia hết cho $3$

 

Do đó $(a+b+c)^3-3(a+b+c)(ab+bc+ca)$ chia hết cho $9$. Điều này vô lý vì theo đk thì $a^3+b^3+c^3$ chia hết cho $9$ nhưng $3abc$ không chia

 

hết cho $9$




#541390 với a,b,c >=0 , a+b+c=1 chứng minh $\sum \sqrt{\...

Đã gửi bởi lahantaithe99 on 20-01-2015 - 00:12 trong Phương trình, hệ phương trình và bất phương trình

với a,b,c >=0, a+b+c=1, chứng minh:

$\sum \sqrt{\frac{ab+2c^{2}}{1+ab-c^{2}}} \geqslant 2+ab+ac+cb$

 

BĐT trên sai với $a=b=0,25$  , $c=0,5$




#541333 ĐỀ THI KIỂM TRA ĐỘI TUYỂN KHTN LỚP 10 VÒNG 2 NĂM 2015

Đã gửi bởi lahantaithe99 on 19-01-2015 - 18:49 trong Thi HSG cấp Tỉnh, Thành phố. Olympic 30-4. Đề thi và kiểm tra đội tuyển các cấp.

Thay $(n,n-1)$ $= > a_{n+1}=7a_{n}-a_{n-1}-2$.Từ đó và từ đề bài 

 

 $= > a_{n+2}-7a_{n+1}+a_{n}=a_{n+1}-7a_{n}+a_{n-1}= > a_{n+2}-8a_{n+1}+8a_{n}-a_{n-1}=0$

 

 Đến đây giải pt đặc trưng là xong

Giải phương trình đặc trưng mặc dù vẫn ra nhưng xong biến đổi rắc rối hơn. Nên sử dụng dãy phụ $(b_n)$ thỏa mãn $b_n^2=a_n$

$b_1=1,b_2=2,b_3=5,b_{n+1}=3b_{n}-b_{n-1}$

Sau đó chứng minh quy nạp sẽ dẫn đến $b_n^2=a_n$




#541321 ĐỀ THI KIỂM TRA ĐỘI TUYỂN KHTN LỚP 10 VÒNG 2 NĂM 2015

Đã gửi bởi lahantaithe99 on 19-01-2015 - 18:01 trong Thi HSG cấp Tỉnh, Thành phố. Olympic 30-4. Đề thi và kiểm tra đội tuyển các cấp.

                     ĐỀ THI KHẢO SÁT ĐỘI TUYỂN LỚP 10 KHTN NĂM 2015

 

Câu I: Giải hệ phương trình

$\left\{\begin{matrix} 2x^3+1=3zx & & \\ 2y^3+1=3xy & & \\ 2z^3+1=3yz & & \end{matrix}\right.$

 

Câu II: Cho dãy $\left \{ a_n \right \}$ ( $n\in\mathbb{N^+}$) xác định bởi 

 

$\left\{\begin{matrix} a_1=1,a_2=4 & \\ a_{n+2}=7a_{n+1}-a_n-2 & \end{matrix}\right.$

 

CMR $a_n$ là số chính phương với mọi số nguyên dương $n$

 

Câu III: Cho tam giác $ABC$ có $M,N$ lần lượt thuộc đoạn $CA,AB$ sao cho $MN$ song song với $BC$. $P$ là điểm di chuyển trên đoạn $MN$. Lấy điểm $E$ sao cho $EP\perp AC$ và $EC\perp BC$. Lấy $F$ sao cho $FP\perp AB$ và $FB\perp BC$

 

a) Chứng minh rằng $EF$ luôn đi qua một điểm cố định khi $P$ di chuyển

 

b) Đường thẳng qua $A$ vuông góc $EF$ cắt $BC$ tại $Q$. CMR trung trực $BC$ chia đôi $PQ$

 

c) Gọi $EM$ cắt $FN$ tại $L$. $AQ$ cắt $MN$ tại $R$. Chứng minh rằng $RL\perp BC$

 

Câu IV Cho đa thứ $P(x)$ thỏa mãn $P(0)=2014!$ và $(x-1)P(x-1)=(x-2015)P(x)$. CMR đa thức $(P(x))^2+1$ không thể phân tích thành tích của hai đa thức với hệ số nguyên có bậc lớn hơn hoặc bằng $1$

 

Câu V: Cho $a,b,c,d>0$ và $a+b+c+d=4$. CMR

 

$P=\frac{(a+\sqrt{b})^2}{\sqrt{a^2-ab+b^2}}+\frac{(b+\sqrt{c})^2}{\sqrt{b^2-bc+c^2}}+\frac{(c+\sqrt{d})^2}{\sqrt{c^2-cd+d^2}}+\frac{(d+\sqrt{a})^2}{\sqrt{d^2-ad+a^2}}\leq 16$

 

                                             ____________________Hết______________________




#540817 Chứng minh: $x^2y+y^2z+z^2x+xyz\le\frac{4}{27...

Đã gửi bởi lahantaithe99 on 14-01-2015 - 19:20 trong Bất đẳng thức và cực trị

Ta sẽ chứng minh $4(a+b+c)^3-27(a^2b+b^2c+c^2a+abc)\geqslant 0$
Xét hiệu: 
$4(a+b+c)^3-27(a^2b+b^2c+c^2a+abc)-4(a+b-2c)^3+27(a-c)^2(b-c)=9c(a^2+b^2+c^2-ab-bc-ca)\geqslant 0$

Do đó khi $c=\text{min}\{a,b,c\}$ thì ta cần chứng minh $4(a-c+b-c)^3-27(a-c)^2(b-c)=(a-c-2b+2c)^2(4a+b-5c)\geqslant 0$ luôn đúng.
Đẳng thức xảy ra khi và chỉ khi $a=b=c$ hoặc $a=2b, c=0$ và các hoán vị tương ứng.

Em có thể nêu ý tưởng cho việc xét hiệu này không? :))